Laurent series $f(z)=frac{sqrt{z}}{z+i}$











up vote
0
down vote

favorite












I came across to calculate an integral for which I had to find the Laurent series of $f(z)=dfrac{sqrt{z}}{z+i}$.



I see that at $z=-i$ the function has a simple pole and the Laurent series I got is as follows:



for $0<|z|<1$ we have $dfrac{sqrt{z}}{i},left(dfrac{1}{1+dfrac{z}{i}} right)=dfrac{sqrt{z}}{i},left(1-dfrac{z}{i}+dfrac{z^2}{i^2}-cdots right)$



and for $|z|>1$ we have $dfrac{sqrt{z}}{z},left(dfrac{1}{1+dfrac{i}{z}} right)=dfrac{sqrt{z}}{z},left(1-dfrac{i}{z}+dfrac{i^2}{z^2}-cdots right)$.



Thus $f(z)=dfrac{sqrt{z}}{i},left(1-dfrac{z}{i}+dfrac{z^2}{i^2}-cdots right)+dfrac{sqrt{z}}{z},left(1-dfrac{i}{z}+dfrac{i^2}{z^2}-cdots right)$.



I am not particularly sure about the answer. Any suggestions or hints would be appreciated.










share|cite|improve this question




















  • 2




    There is no continuous branch of $sqrt z$ in a neighborhood of $0$ so the question does not make sense. Besides you cannot have $sqrt z$ in the terms of any Laurent series.
    – Kavi Rama Murthy
    Nov 15 at 7:29












  • Oh yeah! I get it. Thank you Sir @Kavi Rama Murthy
    – Yadati Kiran
    Nov 15 at 7:31















up vote
0
down vote

favorite












I came across to calculate an integral for which I had to find the Laurent series of $f(z)=dfrac{sqrt{z}}{z+i}$.



I see that at $z=-i$ the function has a simple pole and the Laurent series I got is as follows:



for $0<|z|<1$ we have $dfrac{sqrt{z}}{i},left(dfrac{1}{1+dfrac{z}{i}} right)=dfrac{sqrt{z}}{i},left(1-dfrac{z}{i}+dfrac{z^2}{i^2}-cdots right)$



and for $|z|>1$ we have $dfrac{sqrt{z}}{z},left(dfrac{1}{1+dfrac{i}{z}} right)=dfrac{sqrt{z}}{z},left(1-dfrac{i}{z}+dfrac{i^2}{z^2}-cdots right)$.



Thus $f(z)=dfrac{sqrt{z}}{i},left(1-dfrac{z}{i}+dfrac{z^2}{i^2}-cdots right)+dfrac{sqrt{z}}{z},left(1-dfrac{i}{z}+dfrac{i^2}{z^2}-cdots right)$.



I am not particularly sure about the answer. Any suggestions or hints would be appreciated.










share|cite|improve this question




















  • 2




    There is no continuous branch of $sqrt z$ in a neighborhood of $0$ so the question does not make sense. Besides you cannot have $sqrt z$ in the terms of any Laurent series.
    – Kavi Rama Murthy
    Nov 15 at 7:29












  • Oh yeah! I get it. Thank you Sir @Kavi Rama Murthy
    – Yadati Kiran
    Nov 15 at 7:31













up vote
0
down vote

favorite









up vote
0
down vote

favorite











I came across to calculate an integral for which I had to find the Laurent series of $f(z)=dfrac{sqrt{z}}{z+i}$.



I see that at $z=-i$ the function has a simple pole and the Laurent series I got is as follows:



for $0<|z|<1$ we have $dfrac{sqrt{z}}{i},left(dfrac{1}{1+dfrac{z}{i}} right)=dfrac{sqrt{z}}{i},left(1-dfrac{z}{i}+dfrac{z^2}{i^2}-cdots right)$



and for $|z|>1$ we have $dfrac{sqrt{z}}{z},left(dfrac{1}{1+dfrac{i}{z}} right)=dfrac{sqrt{z}}{z},left(1-dfrac{i}{z}+dfrac{i^2}{z^2}-cdots right)$.



Thus $f(z)=dfrac{sqrt{z}}{i},left(1-dfrac{z}{i}+dfrac{z^2}{i^2}-cdots right)+dfrac{sqrt{z}}{z},left(1-dfrac{i}{z}+dfrac{i^2}{z^2}-cdots right)$.



I am not particularly sure about the answer. Any suggestions or hints would be appreciated.










share|cite|improve this question















I came across to calculate an integral for which I had to find the Laurent series of $f(z)=dfrac{sqrt{z}}{z+i}$.



I see that at $z=-i$ the function has a simple pole and the Laurent series I got is as follows:



for $0<|z|<1$ we have $dfrac{sqrt{z}}{i},left(dfrac{1}{1+dfrac{z}{i}} right)=dfrac{sqrt{z}}{i},left(1-dfrac{z}{i}+dfrac{z^2}{i^2}-cdots right)$



and for $|z|>1$ we have $dfrac{sqrt{z}}{z},left(dfrac{1}{1+dfrac{i}{z}} right)=dfrac{sqrt{z}}{z},left(1-dfrac{i}{z}+dfrac{i^2}{z^2}-cdots right)$.



Thus $f(z)=dfrac{sqrt{z}}{i},left(1-dfrac{z}{i}+dfrac{z^2}{i^2}-cdots right)+dfrac{sqrt{z}}{z},left(1-dfrac{i}{z}+dfrac{i^2}{z^2}-cdots right)$.



I am not particularly sure about the answer. Any suggestions or hints would be appreciated.







complex-analysis proof-verification alternative-proof laurent-series






share|cite|improve this question















share|cite|improve this question













share|cite|improve this question




share|cite|improve this question








edited Nov 23 at 17:45









Martin Sleziak

44.6k7115270




44.6k7115270










asked Nov 15 at 7:27









Yadati Kiran

1,634519




1,634519








  • 2




    There is no continuous branch of $sqrt z$ in a neighborhood of $0$ so the question does not make sense. Besides you cannot have $sqrt z$ in the terms of any Laurent series.
    – Kavi Rama Murthy
    Nov 15 at 7:29












  • Oh yeah! I get it. Thank you Sir @Kavi Rama Murthy
    – Yadati Kiran
    Nov 15 at 7:31














  • 2




    There is no continuous branch of $sqrt z$ in a neighborhood of $0$ so the question does not make sense. Besides you cannot have $sqrt z$ in the terms of any Laurent series.
    – Kavi Rama Murthy
    Nov 15 at 7:29












  • Oh yeah! I get it. Thank you Sir @Kavi Rama Murthy
    – Yadati Kiran
    Nov 15 at 7:31








2




2




There is no continuous branch of $sqrt z$ in a neighborhood of $0$ so the question does not make sense. Besides you cannot have $sqrt z$ in the terms of any Laurent series.
– Kavi Rama Murthy
Nov 15 at 7:29






There is no continuous branch of $sqrt z$ in a neighborhood of $0$ so the question does not make sense. Besides you cannot have $sqrt z$ in the terms of any Laurent series.
– Kavi Rama Murthy
Nov 15 at 7:29














Oh yeah! I get it. Thank you Sir @Kavi Rama Murthy
– Yadati Kiran
Nov 15 at 7:31




Oh yeah! I get it. Thank you Sir @Kavi Rama Murthy
– Yadati Kiran
Nov 15 at 7:31















active

oldest

votes











Your Answer





StackExchange.ifUsing("editor", function () {
return StackExchange.using("mathjaxEditing", function () {
StackExchange.MarkdownEditor.creationCallbacks.add(function (editor, postfix) {
StackExchange.mathjaxEditing.prepareWmdForMathJax(editor, postfix, [["$", "$"], ["\\(","\\)"]]);
});
});
}, "mathjax-editing");

StackExchange.ready(function() {
var channelOptions = {
tags: "".split(" "),
id: "69"
};
initTagRenderer("".split(" "), "".split(" "), channelOptions);

StackExchange.using("externalEditor", function() {
// Have to fire editor after snippets, if snippets enabled
if (StackExchange.settings.snippets.snippetsEnabled) {
StackExchange.using("snippets", function() {
createEditor();
});
}
else {
createEditor();
}
});

function createEditor() {
StackExchange.prepareEditor({
heartbeatType: 'answer',
autoActivateHeartbeat: false,
convertImagesToLinks: true,
noModals: true,
showLowRepImageUploadWarning: true,
reputationToPostImages: 10,
bindNavPrevention: true,
postfix: "",
imageUploader: {
brandingHtml: "Powered by u003ca class="icon-imgur-white" href="https://imgur.com/"u003eu003c/au003e",
contentPolicyHtml: "User contributions licensed under u003ca href="https://creativecommons.org/licenses/by-sa/3.0/"u003ecc by-sa 3.0 with attribution requiredu003c/au003e u003ca href="https://stackoverflow.com/legal/content-policy"u003e(content policy)u003c/au003e",
allowUrls: true
},
noCode: true, onDemand: true,
discardSelector: ".discard-answer"
,immediatelyShowMarkdownHelp:true
});


}
});














draft saved

draft discarded


















StackExchange.ready(
function () {
StackExchange.openid.initPostLogin('.new-post-login', 'https%3a%2f%2fmath.stackexchange.com%2fquestions%2f2999362%2flaurent-series-fz-frac-sqrtzzi%23new-answer', 'question_page');
}
);

Post as a guest















Required, but never shown






























active

oldest

votes













active

oldest

votes









active

oldest

votes






active

oldest

votes
















draft saved

draft discarded




















































Thanks for contributing an answer to Mathematics Stack Exchange!


  • Please be sure to answer the question. Provide details and share your research!

But avoid



  • Asking for help, clarification, or responding to other answers.

  • Making statements based on opinion; back them up with references or personal experience.


Use MathJax to format equations. MathJax reference.


To learn more, see our tips on writing great answers.





Some of your past answers have not been well-received, and you're in danger of being blocked from answering.


Please pay close attention to the following guidance:


  • Please be sure to answer the question. Provide details and share your research!

But avoid



  • Asking for help, clarification, or responding to other answers.

  • Making statements based on opinion; back them up with references or personal experience.


To learn more, see our tips on writing great answers.




draft saved


draft discarded














StackExchange.ready(
function () {
StackExchange.openid.initPostLogin('.new-post-login', 'https%3a%2f%2fmath.stackexchange.com%2fquestions%2f2999362%2flaurent-series-fz-frac-sqrtzzi%23new-answer', 'question_page');
}
);

Post as a guest















Required, but never shown





















































Required, but never shown














Required, but never shown












Required, but never shown







Required, but never shown

































Required, but never shown














Required, but never shown












Required, but never shown







Required, but never shown







Popular posts from this blog

Quarter-circle Tiles

build a pushdown automaton that recognizes the reverse language of a given pushdown automaton?

Mont Emei